« first day (39 days earlier)      last day (535 days later) » 
00:00 - 16:0016:00 - 00:00

4:07 PM
0
Q: Understanding lemma $\left\lVert f \right\rVert_{\infty} = \left|f(a) \right| \leq V_{a}^{b} f$

Von KarI'm learning about functions of bounded variations and need help to understand the proof of this lemma: Lemma. If $f : [a,b] \rightarrow \mathbb{R}$ is of bounded variation, then f is also bounded and satisfies $\left\lVert f \right\rVert_{\infty} = \left|f(a) \right| \leq V_{a}^{b} f$. ...

0
Q: Mapping of the circle |z-1-i|=2^1/2

A.AalipourI have a problem I think it is a little hard or at least has some points need to be considered to solve it. I know that if the transformation function is with the form of fog I should calculate the region with g then claulate the new region with f. But whatever I did, didn't help. The question is...

Questions tend to get more attention when they have a tag for a broad area of mathematics relevant to the question. Some of these tags might fit. (from a bot)Normal Human 21 secs ago
 
4:23 PM
0
Q: Travelling salesman problem as an integer linear program

JamesSo the travelling salesman problem is a problem wherein a salesman has to travel through all cities in a way that the total travelling distance is minimal. You can rewrite this as an integer linear problem: Label the cities with the numbers $0, ...,n$ and define: $x_{ij} = \begin{cases} 1 &

 
1
Q: Would you like to have the option to edit a question without it being bumped to the main page?

gebruikerFor the sake of argument, all edits are considered an improvement in this post. No matter how minor. Since we consider edits an improvement, I feel free to say that they are a good thing. However, one can imagine that not all edits are equally grave. I would like to use the following example:...

 
0
Q: Evaluating Riemann integral using Cauchy criterium

Reda3019Prove using the definition and the Cauchy criterium that $x 􏰀→ x3$ is Riemann integrable on $[a,b]$ and that 􏰁 $\int_a^{b}x^2dx= a^3 - b^3 /3$ Thank you.

0
Q: Simplifying complex equation using polar coordinates

QuestionnaireI should solve this using the polar coordinates, but I just do not know how to start...

0
Q: About closed map

VrouvrouCan someone help me to prove this equivalence : let $f:E \rightarrow F$ we have $1)$ for all $y\in F$ and for all open $U$ from $E$ such that $f^{-1}(\{y\})\subset U$, there exists a neighborhood $V$ of $y$ such that $f^{-1}(V)\subset U$. 2) $f$ is closed I strated by $1) \Rightarrow 2)$ l...

Short title. About closed map
0
Q: sequence and measurable set

ozzyPlease is the following assertion true? Let $\tau\subset\mathbb{R}_+=[0,\infty)$ be a measurable set (the standard Lebesgue measure), $\mu(\mathbb{R}_+\backslash \tau)=0$. Then there exists an increasing sequence $\{x_i\}_{i\in\mathbb{N}}\subset \tau,\quad x_i\to\infty$ such that for any $k,i,\...

Short title. Question contains please. sequence and measurable set
0
Q: link between determinant

tomasI'm a bit confused with this determinant. We have the determinant [picture determinant][1] [1]: http://i.stack.imgur.com/veqk0.png for a matrix $\mathbb{M_n}$ with n$\geq2$. I compute $\delta 2=19$, $\delta 3=65$ Then I would like to find a relation for n$\geq 4$ which links $\delta_n, \delta...

0
Q: Number theory problem RSA encryption

DodgeLet p and q be distinct prime numbers and N = pq. G ={natural numbers less than pq that are relatively prime to pq}. if a and b are in G and k is a natural number, then (1) G has (q-1)(p-1) elements. (2) ab mod N is never 0. (3) ak mod N is never 0. (4) ab mod N and ak mod N are each elements...

Title contains problem. Number theory problem RSA encryption
0
Q: Name for probability distribution function supported on a closed interval and increasing

MaesumiI am looking for names/examples/references for probability distribution functions which are supported on a closed interval, say $[0,1]$, and increasing there.

0
Q: Solve Mathematics puzzle

user297906What is the answer to the following puzzle: If 3 and 2 = 7 5 and 4 = 23 7 and 6 = 47 9 and 8 = 79 then what is 10 and 9 = ?

Short title. Short question. Solve Mathematics puzzle
0
Q: Convex function?

GauchoI have a positive function $f(x,y)$, where $x\in{\mathbb R}^n$ and $y\in{\mathbb R}$. I know that for $y$ fixed, $g(x)=f(x,y)$ is convex, and that for $x$ fixed, $h(y)=f(x,y)$ has positive second derivative. If this enough to show that $f(x,y)$ is convex?

Short title. Convex function?
0
Q: Endomorphism ring of Drinfeld modules.

AndryLet $\mathcal{X}$ be a smooth geometrically irreducible projective curve over $\mathbb{F}_q$. Fix a closed point $\infty\in \mathcal{X}(\bar{\mathbb{F}_q})$. Let $K$ be the function field of $\mathcal{X}$ and $\mathcal{A}$ the ring of functions on $\mathcal{X}$ regular away from $\infty$. Let $\p...

Questions tend to get more attention when they have a tag for a broad area of mathematics relevant to the question. Some of these tags might fit. (autocomment)Normal Human 21 secs ago
0
Q: What is exactly a “Point” ?

Aaryan DewanI read somewhere that a line is made up of infinite points. Between any two points on that line, there are another infinite points. and between any two points BETWEEN those 2 points there are another infinite points. Is this true? If so , HOW? Also, i have read in physics that the smallest p...

0
Q: Find the order of the given elements of the direct product.

sunyata$(1) ([8], [10]) \in Z_{12} \times Z_{18}$ $(2) ([3], [6], [12], [16]) \in Z_4 \times Z_{12} \times Z_{20} \times Z_{24}$

0
Q: why does $A−λ_iI$ lose rank?

lyncrewhy does $A−λ_iI$ lose rank? In other words, why does rank(A) is larger than rank($A−λ_iI$)?

Short title. Short question. why does $A−λ_iI$ lose rank?
0
Q: In $ n < \cfrac{n^2(s-2)-n(s-4)}{2}$ can I make it so that one side becomes completely know

redelectronsIf I have $$ n < \cfrac{n^2(s-2)-n(s-4)}{2}$$ can I make it so that one side becomes completely known when I give a value to $s$, for example(obv. not true) $$123n^2+456n+789<987s+654$$

0
Q: Convexity of varance for large problem

RezlenHow can prove the following optimization model is convex for $n>3$? $\psi (x)= \min \sum_{i=1}^{n} (x_i-\frac{\sum_{j=1}^{n}x_j}{n})^2$

Title contains problem. Short question. Convexity of varance for large problem
0
Q: Math behind gradient tool

Lama SonmezSince math is the most important part for programming I want to implement the gradient tool and I need to understand the math behind it . Can any body also explain the differences between linear , radial ,angle,reflected,diamond gradient? 2 []3 []4

0
Q: Proof that any finite subgroup of SO2 must be cyclic

user18475How do I prove that any finite subgroup of SO2 must be cyclic? Also, what are all the finite subgroups of O2?

Consider adding a tag for a broader subject area to which the question belongs. Some of these tags might fit. (from a bot)Normal Human 21 secs ago
 
5:20 PM
0
Q: Inverse funtion on given sets

Joshua MathewsMy question is: Given sets A = {a1, a2} and B = {b1}, let the function f from A to B be given by the following set of ordered pairs, f = { (a1, b1), (a2, b1) }. If f has an inverse function, call it g, and write g as a set of ordered pairs. If f does not have an inverse function, explain why it d...

0
Q: Prove that a polynomial which is identically zero in some open subset of a Banach space is zero in the whole space.

AdolfoI'm reading Lemma 1.3.8 of Mujica's "Complex Analysis in Banach Spaces" and I'm having some trouble understanding the proof of it. The lemma states: Let $f: E \to F$ be a mapping such that $f(a + \lambda b)$ is a polynomial in $\lambda$ for all $a$, $b \in E$. If $f$ is identically zero on a ...

 
-1
Q: Why was my question on a php.net-comment downvoted?

simonthesorcererI asked a question here. As you can see, noone really liked it, and people voted to close it as off-topic. I don't have a problem with this, I only want to understand why. In my question, I ask for the explanation of a comment on php.net. The first comment to my question is that the comment re...

0
Q: User Participation in Help and Improvement Review Queue

HaveNoDisplayNameI mostly active on Reviews on SO. Since last 4-5 months, I noticed that total number of daily reviews done in "Help and Improvement" are degrade than other review queues on daily basis. When "Help and Improvement" review queue launched, then more users take actively participate in Edit/Improve ...

 
0
Q: Matrices representing symmetries

user18475Is there a direct way to find the matrices representing the symmetries for example of a tetrahedron with vertices (1 1 1) (-1 -1 1) (-1 1 -1) (1 -1 -1)

Short title. Short question. Matrices representing symmetries
0
Q: how to prove a formula is not valid in robinson arithmetic

Chen LuI have known that ∀x [s(0)∗ x = x] is a theorem of Peano Arithmetic. but how to Prove that this formula is not a theorem of Robinson Arithmetic?

0
Q: How to obtain $K$ in $K[1,2]$ in Maple?

LJRI would like to obtain the letter $K$ from $K[1,2]$ in Maple. I tried to use op(K[1, 2]). But I get $[1,2]$. Are there some function $f$ in Maple such that $f(K[1,2]) = K$? Any help with be greatly appreciated!

Tagged maple. [How to obtain $K$ in $K[1,2]$ in Maple?](math.stackexchange.com/q/1571128)
0
Q: Sources estimation for identification of anomalous events

DarioI’m stuck on the following problem. There are two sources $S_A$ and $S_B$ at the ends of a channel. Both are made up of a white noise component $W_i$ plus an impulsive component $I_i$: $S_A = W_A + I_A$; $S_B = W_B + I_B$. Two sensors are co-located with the sources. The first one records the...

Questions tend to get more attention when they have a tag for a broad area of mathematics relevant to the question. Some of these tags might fit. (from a bot)Normal Human 21 secs ago
0
Q: Solve the given Differential Equation

MathematicsSolve the equation $\frac{dy}{dy}=\frac{x+2y-5}{2x+xy-4} $ I tried substituting $x=X+h$ and $y=Y+k$ but the $xy$ term is creating problem. How to solve it?

0
Q: $\mathbb{R}/\mathbb{Z}\cong S^1$

sequence$S^1 := \{z\in \mathbb{C}: |z|=1\}$ is the set of roots of unity. We can use the fact discovered by Euler that any complex number $z$ can be represented as $z=re^{\pi i k}$, where $r\in \mathbb{R}$ is the radius of $z$ and $k$ is any real number. Any complex number with radius $1$ can thus be def...

A title should not be all-MathJax; having some plain text helps with search and navigation. (from a bot)Normal Human 21 secs ago
0
Q: Show covariance matrix must have condition number larger than correlation matrix

IvanaThe inverse of covariance matrix can be used to find the conditional independencies among variables. This inverse is more sensitive to changes then the correlation matrix. As a result two samples with very similar correlation matrices and standard deviations, may still point to different conditio...

0
Q: number theory RSA encryption problem 2

DodgeLet p and q be distinct prime numbers and N =pq. Do multiplication mod N. Define G = {natural numbers less than pq that are relatively prime to pq} and s =(p-1)(q-1) . Then (1) m^s =1 for all m in G. I have no clue with first part of this problem, and don't know how to start with it.

Title contains problem. Title ends with a digit. number theory RSA encryption problem 2
This site uses MathJax formatting of formulas. Questions tend to get more attention when they have a tag for a broad area of mathematics relevant to the question. Some of these tags might fit. More tips here. (from a bot)Normal Human 24 secs ago
0
Q: A question regarding Bayes Naive Classification

alkabaryI do understand the concept of Naive Bayesian classification, as it tries to calculate the probability of an outcome of a class given multiple evidences. It comes from the Bayes theorem and it is called naive as it tries to each of piece of evidence as independent. This approach is why this is ca...

Words such as question do not add information to titles. Please edit the title so that it better describes the specifics of your question. Do not hesitate to make it longer or include a formula if needed. More tips here. (from a bot)Normal Human 21 secs ago
0
Q: prove E(X1+X2|Y)=E(X1|Y)+E(X2|Y)

Daniel Liprove E(X1+X2|Y)=E(X1|Y)+E(X2|Y) This question is given in the text book as an exercise. It looks trivial but took me hours to think of a proof. I don't know what to do after expand it using definition of expectation...

0
Q: Find the Area of the circle.

DCoder Objective:To find r and area of the circle, when two parameters are given.

Short title. Short question. Find the Area of the circle.
 
6:08 PM
0
Q: What is the fundamental group of $RP^2\#T$, the real projective space of dimension 2 and T is a torus?

grayQuantI would like to see how this is done, probably using van Kampen. How to see this space?

0
Q: Show that $(A')'\subset A'$ is not always true.

mahdi moosazadehAsumme that $(S,T) $ is topology space and $A'$ is set of limit pointso of set $A$ show that $(A')'\subset A'$ is not always true. I can't find a example to show that.

0
Q: evaluating Riemann integral using definition and cauchy criterium

Reda3019This is a question in one of the previous exams in my introductory real analysis course. The question wants us wants to use the definition of the riemann integral and cache's criterium to prove that $x^2$ integrable over $[a,b]$ and prove that $\int_a^b x^2 dx = \frac{b^3 -a^3}{3}$.

Consider replacing (analysis) with a more specific tag for the relevant branch of analysis. (from a bot)Normal Human 20 secs ago
0
Q: Show that $rank A = 3$

Joe MahanashaMatrix $A \in R^{3, 2015}$ is given. It is known that matrix $AA^{T}$ is invertible. Show that $rank A = 3$. How to start this? What does the info that $AA^{T}$ is invertible gives us?

Short title. Short question. Show that $rank A = 3$
0
Q: Maxima and minima of a two variable function

VenkatFind the maxima, minima and saddle points of the surface Z = (x^2 -y^2)e^((-x^2 -y^2)/2)

This site uses MathJax formatting of formulas. More tips here. (from a bot)Normal Human 20 secs ago
0
Q: Composition of sets

Joshua MathewsQuestion: Let the set A be defined as A = {a, b, c, d}, and let the relations R and S on the set A be defined as R = {(d, a), (a, b), (b, c)}, and S = {(a, a), (b, d), (d, c)}. Explain why the ordered pair (a, b) is or is not an element of the composition of S and R (denoted R o S). I would ...

Short title. Composition of sets
0
Q: How to find the intersection of two spaces

Jade clarke 2208For spaces X=(t,2t-s,s,3s-t) Y=(2p-q,p-q,3p-q,5q) find their intersection and show that their intersection is a 1-dimensional subspace of R^4. Thanks :)

Welcome to Math.SE, Jade clarke 2208. Questions tend to get more attention when they have a tag for a broad area of mathematics relevant to the question. Some of these tags might fit. (from a bot)Normal Human 20 secs ago
0
Q: Logs with exponential bases

theBIOguyI know that e^[(log base e^2) of 16] is 4, but I can only get this far: e^[(log base e^2) of 4^2] e^2[(log base e^2) of 4] I need some way to cancel the 2's so I can get e^[log base e of 4] but I don't the identity or rule that cancels the 2's.

0
Q: Problem with integral representation of a probability measure

KolminHere I have a rather naive question concerning integral representation of probability measures. In general I have problems with it, so here there is a super basic setting: $(X, \Sigma, \mu)$ probability space, $A, B \in \Sigma$, $\chi_B \in [0,1]^X$ indicator function of $B$ (measurable), $\i...

This site uses MathJax formatting of formulas. More tips here. (autocomment)Normal Human 20 secs ago
 
6:49 PM
0
Q: Midpoint rule from table

Madelyn ShroederCompute R4; L4; M2; M4 where M2 is the midpoint rule with 2 rectangles. M4 is midpoint with four rectangles. Notice you'll have to estimate with M4. x 0 2 4 6 8 y 5 6 8 11 14

Short title. Short question. Midpoint rule from table
0
Q: Quantificational Logic: "For every number a, the equation $ax^2 + 4x -2 = 0$ has at least one solution."

user242007This is a solution check for my quantified representation of the following statement. "For every number a, the equation $ax^2 + 4x -2 = 0$ has at least one solution." $\forall a[$iff $a \ge 2$ there exists at least one solution such that $ax^2 + 4x -2 = 0]$ $\forall a[(a \ge 2) \leftrightarrow...

0
Q: Find limit for $n*C^n$ when $|C|<1$

eontorchFind $\lim_{n\to \infty}{n{C^n}}$ when: $|C|<1$ I want to use the squeeze theorem so I bounded it from below with: $C^n\to 0$ But I can't find the upper bound.

0
Q: kinetic energy interation

kprincipeI was reading the following article http://www.askamathematician.com/2015/03/q-why-does-kinetic-energy-increase-as-velocity-squared/ I don't understand the math of the explanation at the final of the article. The author explain what he is using on every step, for example if it is the chain rule. ...

0
Q: Midpoint rule and natural log

Madelyn ShroederRecall that ln x = We are going to estimate ln 4. Use 6 rectangles and midpoint rule.

Short title. Short question. Midpoint rule and natural log
0
Q: Calculating coefficients in a differential equation

martin1982It's a pretty open-minded exercise I found online. It says, you're advising a social network company and they're trying to model an equation for $u(t)$, being this the amount of active users in the network. They know their equation will have this following "shape". $$\frac{\partial^2 u}{\partial...

Tagged differential-equations but mentions "partial". Calculating coefficients in a differential equation
0
Q: Vertex sets bijection fuction

Joshua MathewsLet vertex sets V1 and V2 be defined by V1= {1, 2, 3} and V2 = {a, b, c}. Let E1 = { { 1, 2}, {2, 3} }, and let E2 = { {a, b}, {b, c} } be the edge sets corresponding to the vertex sets V1 and V2, respectively. Write a function f that is a bijection from V1 to V2. a. Write your function f as a...

1
Q: nth root function

ZelphirI want to write code for a nth root function, so I need to be sure, that the underlying mathematical function is correct. From another post over at SO, I wrote the following definition: $ \sqrt[x]{y} = y^{\frac{1}{x}} = \left\{ {\begin{array}{rl} \exp_{2}\left(x \cdot \log_2 \left(\frac{1}{y}\ri...

Short title. nth root function
0
Q: To what is that equivalent?

Mary StarSuppose we have the system $$\exists y (G(y)=f \land G_1(y)\neq g_1 \land G_2(y)\neq g_2).$$ Is this equivalent to $$\not\exists y \left (G(y)=f \land G_1(y)= g_1 \land G_2(y)= g_2) \land \exists x(G(x)=f\right )$$ ? Or to $$\not\exists y_1, \not\exists y_2 \left ((G(y_1)=f \land G_1(y_1)= g...

 
0
Q: Why aren't the Suffrage and Vox Populi badges paired together on the badge page?

AHigginsI earned the Vox Populi badge earlier today, which prompted me to read over the badge page in the Help Center and see what else I might be close to earning get confused about the Suffrage badge I already have read a dozen meta posts on why we have both Vox Populi and the Suffrage badge While...

 
7:19 PM
0
Q: Why are trivial solutions "wrong"?

JimmWhy are trivial solutions "wrong"? For example, if I'm solving a PDE and the eigenvalue being zero implies that the solution to the PDE is identically $0$, why do we say that the eigenvalue cannot equal $0$?

 
0
Q: Code not rendered properly

Jacques MaraisI have put a bounty on a question a few days ago. I then came across a rendering problem. As you can see here, the code is not rendered properly, but is written properly as markdown: I have tried editing it myself, but it still doesn't seem to render correctly. I even tried viewing it in Googl...

 
0
Q: How to determine stability of the equilibrium?

user158013Let $ f : \mathbb{R} → \mathbb{R} $ be $C^∞$ suppose the following: (i) $f(0) = 0,$ (ii) there is a smallest $n \in \mathbb{N} $ so that $f^{n}(0) \neq 0$ Determine stability of the equilibrium at $0$ for the differential equation $ \dot x = f(x) $ in terms of $n$ and the sign of $f^{n}(0)$....

1
Q: There exists an algebraic basis $(e_i)_{i \in I}$ in $E$ such that $\|e_i\| = 1$ for all $i \in I$?

Analysis CabalLet $E$ be a infinite-dimensional normed vector space. How do I see that there exists an algebraic basis $(e_i)_{i \in I}$ in $E$ such that $\|e_i\| = 1$ for all $i \in I$?

0
Q: How do I find supremum of this set?

M.RitzI had a test today and I had to find sup and inf of this set :$$ A=\{sin(\frac{nπ}{3}+\frac{1}{n}), n \in N\} $$ I calculated the value of sin for 6 cases (n=6k, n=6k+1, n=6k+2,...) and then said that the minimum value found is the inf and the maximum value is sup. Is this approach correct? If no...

Welcome to Math.SE, M.Ritz. Consider adding a tag for a broader subject area to which the question belongs. Some of these tags might fit. (from a bot)Normal Human 21 secs ago
0
Q: Find the covariance of X and Y given Z = 3x - 2y and the standard deviation of x, y and z.

Will KavanaghThis is the exact problem: `Suppose that X, Y are random variables with Sx =2, Sy = 3. Let Z = 3X - 2Y, and assume that Sz = 6. Find the covariance, cov(X, Y).'

Questions tend to get more attention when they have a tag for a broad area of mathematics relevant to the question. Some of these tags might fit. (from a bot)Normal Human 21 secs ago
0
Q: Inflection point of $y=-x^3$ and $y=\frac{4x(x^2+3)}{(x^2-1)^3}$

DavidMIs it true that funciton $$y=-x^3$$ has inflection point and function $$y=\frac{4x(x^2+3)}{(x^2-1)^3}$$ hasn't inflection point? If yes why??

0
Q: $X$ is exponentially distributed $\varepsilon(\theta)$. Using the Method of Maximum likelihood find the best (marking?)Question its centeredness

Bozo Vulicevic$X$ is exponentially distributed $\varepsilon(\theta)$. Using the Method of Maximum likelihood find the best (marking?)of sample $n$ for parameter $\theta$ .Question its centeredness and existence. Now I think I do not know the exact english translations of these notions so I will explain using t...

0
Q: Solve initial value problem 1st order DE.

stellarhawk 34ive been struggling to solve the following equation. t³*dx/dt + 3*t²*x = t I tried to use its characteristic equation and got the one root for x must be Ae^(-3t/x) but i just end up confusing myself even more. I read that you can do it by separating the variables but im not sure if this ...

This site uses MathJax formatting of formulas. More tips here. (autocomment)Normal Human 21 secs ago
0
Q: Supposed g'(x)=f(x) for all x. Evaluate:Evaluate the integral from [1,e]?

user295692The integral from1,ef(e^x+lnx)dx Also, what are these type of questions called?

Short question. [Supposed g'(x)=f(x) for all x. Evaluate:Evaluate the integral from [1,e]?](math.stackexchange.com/q/1571287)
0
Q: Solving (n+1)(n+2)…(n+k)−k = x^2

villenLet n and k be integers. Need to find all pairs of (n,k) such that (n+1)(n+2)…(n+k)−k = x^2 , where x^2 is a perfect square.

Welcome to Math.SE, villen. This site uses MathJax formatting of formulas. More tips here. (from a bot)Normal Human 21 secs ago
0
Q: Is the Period Symbol (.) Ever Used as an Operator

MCTaylor17Are there any cases in which x.y (where x and y are both integers) would produce some kind of result or be interpreted as anything other than a decimal number? This question relates to this MathML question asked on StackOverflow.

Tag (notation) should not be the only tag a question has. Please add a tag for a subject area to which the question belongs. (autocomment)Normal Human 21 secs ago
0
Q: Cesaro Means convergence

SelfStudyShow that if $\lim x_n =x$ with $x_n \in \mathbb{R}$ and $x \in \mathbb{R}$, then the sequence given by the averages $y_n = \frac{x_1 +x_2 + \dots + x_n}{n}$ also converges to x. Solution: Denote the limit of $(x_n)$ as $a$. Let $\epsilon >0$ Then there exists a positive integer $N_1 > 0$ such...

0
Q: Let a and b be 3D vectors. Which of the following expressions make sense?

GhostLet a and b be 3D vectors. Which of the following expressions make sense? A. (a•b)+a B. (axb)+a C. (axb)•a D. (axb)xa

0
Q: How can I translate this affine hyperplane?

Brian FitzpatrickLet $\mu\in\Bbb Z^d$. Suppose that $H=\{x\in\Bbb Z^d:\langle\mu,x\rangle=1\}$ is nonempty and fix $v\in H$. Now, let $\{a_1,\dotsc,a_{d-1}\}$ be a basis for the kernel of the map $\langle \mu,-\rangle:\Bbb Z^d\to\Bbb Z$. Form a $d\times d-1$ matrix $A$ with these vectors $$ A=\begin{bmatrix} a_1 &

 
7:52 PM
1
Q: StackOverflow Questions Not Accessible - Error 522

David C. RankinI'm not sure where to report this, but if you are not already aware, the StackOverflow cite is unable to respond when a user question is clicked. Additionally the right-side-bar is loading some 10 seconds after the main page load. Cloudfair shows good connection from my ISP to Dallas, TX, but th...

 
Consider adding a tag for a broader subject area to which the question belongs. Some of these tags might fit. (autocomment)Normal Human 21 secs ago
0
Q: Projection of Range of Matrix A onto Kernel of A-I

Yunus SyedLet A be a 2 x 2 matrix such that A^2 = A. Show that Ax = x for every x in R(A) and if rank(A) = 1, M = R(A), N = Ker(A-I) then A is the projection along N onto M. For the first part, I said that A^2 * x = A * x so multiplying by A^-1 on both sides we have A * x = x. This doesn't sound right b...

This site uses MathJax formatting of formulas. More tips here. (autocomment)Normal Human 21 secs ago
0
Q: Does this Laplace's equation have a stable soultion for some interval around $t$?

BR PahariHow do I show that for $f \in C^{\infty}(T)$ for which the Laplace's equation $u_{tt}+ u_{xx} = 0$ with initial conditions $u(x,0)=f(x)$ and $u_t(x,0) =0$ has no soultion with $u(\cdot,t) \in L^2(T)$ in any interval $|t| < \epsilon$ of any $\epsilon >0$. Do I need to find a general solution a...

0
Q: calculating square roots

SelfStudyLet $x_1 = 2$, and define $x_{n+1} = \frac{1}{2} (x_n + \frac{2}{x_n})$ Show that $x_n^2$ is always greater than or equal to 2, and then use this to prove that $x_n − x_{n+1} ≥ 0$. Conclude that $\lim x_n = √2$. My question: So I know how to do this problem but I don't know how to prove that ...

0
Q: Will somebody please give an evaluation of the following integral: ∫ from 0 to 8 of (20 sin(t^2/35)

Ted B. I understand that it is a Fresnel Integral but I don't understand how to evaluate it or find its antiderivative.

 
8:25 PM
0
Q: How to evaluate integral with radian limits?

Analytic LunaticI'm having trouble with formatting (lower limit is (pi/3) and upper limit is (pi/6)): \begin{align} \int csc \theta cot \theta \ \mathrm d\theta &\end{align}

0
Q: Sum of shifted exponential random variables

Suthakaranf(x) = e^-(x-1) ; x>1 I need the sum of above shifted exponential distribution from 1 to infinity.

0
Q: Cardinality of all rational points in $R^3$

MatthewQuestion: Find the cardinality of the set of all points in $R^3$ all of whose coordinates are rational, and justify the answer. Idea: Call the set of all points in $R^3$ all of whose coordinates are rational A. Also, I will make a set called B, that is the set of all coordinates (x,0,0) where x...

0
Q: Question with Divisibility proof

Bob GonrI have a simple proof question: Suppose $a,b \in \Bbb Z$ where $a|b$. If $a|(b-c)$, then $a|c$. I have solved it below, but is my way a valid answer? Is there a better clear way of proving this? Suppose $a|(b-c)$ is true given $a|b$. Then $\frac{b}{a} = k, k \in \Bbb Z. $ Re-arrange for b: $...

Welcome to Math.SE, Bob Gonr. Words such as question do not add information to titles. Please edit the title so that it better describes the specifics of your question. Do not hesitate to make it longer or include a formula if needed. More tips here. (autocomment)Normal Human 21 secs ago
0
Q: descending smoothness along unramified morphisms

user65187Let $f:X\rightarrow Y$, and $g:Y\rightarrow Z$ be morphisms of schemes locally of finite type. Suppose that $f$ is unramified, $g$ is flat and $g\circ f$ is étale. Does $g$ is also étale?

0
Q: Needs some hints regarding Fourier series

David LundDetermine the fourier series for the function defined by: f(x) = 2x ................. 0 <_ x <_ 2pi f(x+2pi) = f(x) I have three questions. Is A0 = 0? Is An = 0? Is Bn = (Something =/ Not 0). This has been my approach so far. I can't seem to get the right answer so I have to root out what ...

This site uses MathJax formatting of formulas. More tips here. (autocomment)Normal Human 21 secs ago
 
8:48 PM
0
Q: Explaining correlation matrix calculations

RLearnsMathHere is given table showing correlation coefficients for given variables: Сorrelation coefficients So I need to take simple calculation: enter image description here

0
Q: Trouble finding the upper bound for a certain sum.

micha987123I have encountered the following problem and I am curious how to solve it. $\textrm{Given }a_{n+1} = a_{n}(1 - \sqrt{a_{n}}) \textrm{, where } a_{i} \in (0,1) \textrm{, } i = \overline{1,n}$ I have proved that $(a_{n})_{n\in\mathbb{N}}$ is decreasing and now I have to prove that the upper bound...

 
9:20 PM
0
Q: Eliminating square root

Kamel IsaacI have a quantity $f=\sqrt{(a+b)^2-c}$ I need to know the explicit dependence of $f$ on the quantity $(a+b)$, is there any way of removing the term $-c$ from the square root and then compensate for it in a subsequent step?

0
Q: Minimizing over permutations

JasonFix $a=(a_1,...,a_n)^T,b=(b_1,...b_n)^T \in \mathbb{R}^n$. Assume WLOG $a_1\geq...\geq a_n$, $b_1 \geq ... \geq b_n$. Let $s$ be a permutation of the indices $\{ 1,...,n \}$. Intuitively, the way to minimize $\sum_{i=1}^n (a_i-b_{s(i)})^2$ would be to choose $s$ to be the identity permutation....

0
Q: Why is $\int_0^L [cos(\frac{n \pi x}{L} - \frac{m \pi x}{L}) - cos(\frac{n \pi x}{L} + \frac{m \pi x}{L})]dx = L $?

JimmWhy is $\int_0^L [cos(\frac{n \pi x}{L} - \frac{m \pi x}{L}) - cos(\frac{n \pi x}{L} + \frac{m \pi x}{L})]dx = L $ ? $n$ is an integer, $L$ is a constant.

Short question. [Why is $\int_0^L [cos(\frac{n \pi x}{L} - \frac{m \pi x}{L}) - cos(\frac{n \pi x}{L} + \frac{m \pi x}{L})]dx = L $?](math.stackexchange.com/q/1571384)
0
Q: Is the localization of a f.g module at a f.g multiplicative monoid a f.g module?

ExteriorLet $M$ be a f.g $R$-module for $R$ commutative. Let $S$ be a multiplicative submonoid of $R$ which is f.g as a monoid. Is it true that $S^{-1}M$ is a f.g $S^{-1}R$-module?

0
Q: Order of the zero

IllustionistSuppose we consider the following function $f(z) = 1 - e^z$ We want to determine the order of the zeroes of this function here. My professor wrote that the order of zero of this function is actually 1? how did he determine that ? Can someone explain?

Short title. Order of the zero
0
Q: Price optimization for maximizing revenue

JuliaAnyone, please explain me what am I doing wrong? I have a data (16 districts, 1 time horizon): Sales volume (number of units) Price per unit Sales __ Price __ Revenue 81996 __ 49 __ S*P 91735 __ 49 __ S*P ....and so on I have to find an elasticity in order to define what is ...

Welcome to Math.SE, Julia. This site uses MathJax formatting of formulas. More tips here. (autocomment)Normal Human 21 secs ago
0
Q: What rule is used for this simplification?

kecman$$ \frac{8}{(s+1)^2 + 2^2} * \frac{1}{s} = \frac{8}{5} - \frac{1}{s} + \frac{16}{10}* \frac{s+1}{(s+1)^2 + 2^2} + \frac{8}{10}* \frac{2}{(s+1)^2 + 2^2} $$

0
Q: inclusion of f.g submodules and localization

ExteriorSuppose $M,N$ are submodules of some other module and that $M$ is f.g. Suppose $S^{-1}M\subset S^{-1}N$. How can I prove $M[\frac 1s]\subset N[\frac 1s]$ for some $s\in S$?

 
9:48 PM
0
Q: Is recurrent sequence a(n+1)=arcsin(a(n) ) converging?

Boris ModelA recurrent sequence a(n+1)=arcsin(a(n)), a(1) belongs to [-1,1], is given. Is it converging?

0
Q: A group with a prime number of elements with prime order?

TheNotMe If $p$ is a prime number, show that there is no group with $p$ elements of order $p$.

0
Q: Franklin Function formula explanation

TermiFrom the paper: https://drive.google.com/file/d/0B8-0WXhCnPn3Vnk1RGpseGJhV28/view In chapter "2.1 Construction of Franklin Functions" there are 2 mathematical formulas. First formula is $φ_i(x) = (x - a_i)_+ , i = 1,2,3$ ... Second formula is $φ_0(x) = 1$, $φ_1(x) = √3 * (2x-1)$, $φ_2(x) = ....

Welcome to Math.SE, Termi. Consider adding a tag for a broader subject area to which the question belongs. Some of these tags might fit. (autocomment)Normal Human 20 secs ago
0
Q: Is every non countable subset of $\mathbb{R}$ second category in $\mathbb{R}$?

Quique RuizLet's consider $\mathbb{R}$ with the absolute value metric. Is every non countable subset of $\mathbb{R}$ second category in $\mathbb{R}$?

0
Q: Curl F and conservative vector field.

Philip ParkIf curl F = 0, is F conservative? I know F conservative implies curl F = 0, however, I want to know if it works the other way.

0
Q: explicit formula for $ _2F_2(2,2;1;1;z) $

Z. AlfataIs it an explicit formula for $$ _2F_2(2,2;1;1;z) $$ thanks you in advance

0
Q: baby Rudin exercise 4.26 on compactness and uniform continuity

somedudeRudin exercise 26 chapter 4: "Suppose $X, Y, Z$ are metric spaces, and $Y$ is compact. Let $f$ map $X$ into $Y$, let $g$ be a continuous one-to-one [not explicitly stated in the book: $g$ need not be onto] mapping of $Y$ into $Z$, and put $h(x)=g(f(x))$ for $x∈X$. Prove that $f$ is uniformly c...

0
Q: Unique real number $c$ such that $f'(x)=cx$ for some point $x$

EvanHow can I show that for a function $f$ that is differentiable on $[0,1]$ st $f(0)=0$ and $f(1)=1$ there is unique $c \in \mathbb{R}$ and some point $x \in (0,1)$ so that $f'(x)=cx$?

0
Q: Puzzle math questions!

Jean PaulCan someone help me with the following puzzle problems?: There is a number with 6 different digits, if we pick the last digit of that number and place before that number we got $5$ times our number; James is two times older than John was when James was as old as John is now, James is 28 years o...

0
Q: (Collatz) Modulo 18 Partitions of Collatz 3n+1 Trajectories

frogfanitwI have examined partial Collatz 3n+1 trajectories going from one odd integer to the next. These lead to an infinite number of repeated patterns where the "next" odd integer is congruent to one of only six patterns: {5, 11, 17, 1, 7 or 3} mod 18. I have formulae for these patterns in terms of ...

0
Q: Friends in a queue

EnjoyLifeWe have a queue of N humans. At this queue there are two friends. What is the probability that between friends will be M humans. (M + 2 < N) So, what we got: total number of combinations = N!; Lets take some example: N=10, M=3. For that case, between friends can be 3 humans, any humans, so...

Short title. Friends in a queue
Welcome to Math.SE, frogfanitw. Consider adding a tag for a broader subject area to which the question belongs. Some of these tags might fit. (from a bot)Normal Human 21 secs ago
0
Q: What is the value of x in this diagram?

Atoosahttp://i.stack.imgur.com/NFnZt.png So I'm pretty familiar with SOH-CAH-TOA but this question in particular looks a bit different and I'm not sure how to go about it. Thanks in advance!

0
Q: Probability for Sum > 9 with Roll of Two Standard Dice

MathBunnyWhat is the probability of a total greater than 9 in a given roll with two standard dice?

 
10:48 PM
0
Q: Challenging problem: IVT/MVT Prove that the function has exactly 2 roots?

user295692Let a,b,c, and d be real numbers. If d < 0 and 3a^2 < 8b, show that x^4+ax^3+bx^2+cx+d = 0 has exactly 2 roots. I know that you have to use IVT to prove that there are atleast 2 distinct roots, but I don't really know how to go about it.

This site uses MathJax formatting of formulas. More tips here. (from a bot)Normal Human 21 secs ago
 
10:58 PM
0
Q: Help proving that scalar multiplication is continuous

GustavoLet $\mathbb{K} \in \{ \mathbb{R} , \mathbb{C} \}$ and $s= \mathbb{K}^{\omega}$ be the usual sequence set with entries on $\mathbb{K}$. I proved that $\mathbb{K}$ induces a $\mathbb{K}$-vector space structure on $s$ and that the function $\rho : s \times s \to \mathbb{R}$ given by $$\displaystyle...

Words such as help do not add information to titles. Please edit the title so that it better describes the specifics of your question. Do not hesitate to make it longer or include a formula if needed. More tips here. (autocomment)Normal Human 21 secs ago
0
Q: How to negate this statement for a proof by contradiction

RustyStatisticianI want to try and construct a proof by contradiction but am having a hard time negating this statement. The statement that I am working with is There are only a finite number of points accepted into the set and this finite sequence converges to a stationary point. So I want to prove th...

Tag (proof-writing) should not be the only tag a question has. Please add a tag for a subject area to which the question belongs. (from a bot)Normal Human 21 secs ago
0
Q: mean value theorem inequality homework

Spongebob Squareroot-pants So I think the interval for this question is (n,n+1) and the function is y=lnx, based what the inequality looks like, but I don't know how to approach this question the "proper" way. Also I am not sure what the (n+1)/n does. so if someone can explain it I would be very happy. Thanks

Title contains homework. mean value theorem inequality homework
0
Q: Age puzzle problem

Jean PaulCan someone help me with the following puzzle problem? I really don't get it: James is two times older than John was when James was as old as John is now, James is 28 years old, how old is John; Thanks in advance!

Short title. Title contains problem. Age puzzle problem
 
11:16 PM
0
Q: Probability / Set Theory Question (Independent/Dependent Event)

MathBunnyI am looking for arguments to support that this is independent. A study on the effects that listening to loud music through headphones had on teenager's hearing found that 12% of those teenagers in the sample who did listen to music in this way showed the signs of hearing problems. If 60% of the...

0
Q: Nominal Rates/Effective rate computation, confusion.

HeavyWeightGiven a nominal rate of 6% per annum. Change it to an effective rate per month. What I do is: $$(1+\frac {0.06}{12})^{12}=(1+i)^{12}$$ where $i$ is the effective interest rate per month. Now what if the question had said that it was the effective annual rate that was 6%, what would I do then? ...

 
11:48 PM
Welcome to Math.SE, ronc22. This site uses MathJax formatting of formulas. More tips here. (from a bot)Normal Human 21 secs ago
 
00:00 - 16:0016:00 - 00:00

« first day (39 days earlier)      last day (535 days later) »